SlideShare a Scribd company logo
Section 5.5
          Integration by Substitution

                  V63.0121.006/016, Calculus I

                         New York University


                         April 27, 2010


Announcements
   April 29: Movie Day
   April 30: Quiz 5 on §§5.1–5.4
   Monday, May 10, 12:00noon Final Exam:
       SILV 703 (Section 16)
       MEYR 121/122 (Section 6)
                                               .   .   .   .   .   .
Announcements



          April 29: Movie Day
          April 30: Quiz 5 on
          §§5.1–5.4
          Monday, May 10,
          12:00noon Final Exam:
                 SILV 703 (Section 16)
                 MEYR 121/122 (Section
                 6)




                                                                               .   .   .     .       .      .

V63.0121.006/016, Calculus I (NYU)   Section 5.5 Integration by Substitution               April 27, 2010       2 / 38
Resurrection Policy
    If your final score beats your midterm score, we will add 10% to its
    weight, and subtract 10% from the midterm weight.




                                                                                       .   .   .     .       .      .
.
Image credit: Scott Beale / Laughing Squid
  V63.0121.006/016, Calculus I (NYU)         Section 5.5 Integration by Substitution               April 27, 2010       3 / 38
Objectives



          Given an integral and a
          substitution, transform the
          integral into an equivalent
          one using a substitution
          Evaluate indefinite
          integrals using the method
          of substitution.
          Evaluate definite integrals
          using the method of
          substitution.



                                                                               .   .   .     .       .      .

V63.0121.006/016, Calculus I (NYU)   Section 5.5 Integration by Substitution               April 27, 2010       4 / 38
Outline



Last Time: The Fundamental Theorem(s) of Calculus


Substitution for Indefinite Integrals
  Theory
  Examples


Substitution for Definite Integrals
  Theory
  Examples




                                                                               .   .   .     .       .      .

V63.0121.006/016, Calculus I (NYU)   Section 5.5 Integration by Substitution               April 27, 2010       5 / 38
Differentiation and Integration as reverse processes


Theorem (The Fundamental Theorem of Calculus)

  1. Let f be continuous on [a, b]. Then
                                 ∫ x
                              d
                                     f(t) dt = f(x)
                             dx a

  2. Let f be continuous on [a, b] and f = F′ for some other function F.
     Then                 ∫                b
                                               f(x) dx = F(b) − F(a).
                                       a




                                                                               .   .   .     .       .      .

V63.0121.006/016, Calculus I (NYU)   Section 5.5 Integration by Substitution               April 27, 2010       6 / 38
Techniques of antidifferentiation?


So far we know only a few rules for antidifferentiation. Some are
general, like
              ∫                    ∫            ∫
                [f(x) + g(x)] dx = f(x) dx + g(x) dx




                                                                               .   .   .     .       .      .

V63.0121.006/016, Calculus I (NYU)   Section 5.5 Integration by Substitution               April 27, 2010       7 / 38
Techniques of antidifferentiation?


So far we know only a few rules for antidifferentiation. Some are
general, like
              ∫                    ∫            ∫
                [f(x) + g(x)] dx = f(x) dx + g(x) dx

Some are pretty particular, like
                   ∫
                           1
                       √         dx = arcsec x + C.
                      x x2 − 1




                                                                               .   .   .     .       .      .

V63.0121.006/016, Calculus I (NYU)   Section 5.5 Integration by Substitution               April 27, 2010       7 / 38
Techniques of antidifferentiation?


So far we know only a few rules for antidifferentiation. Some are
general, like
              ∫                    ∫            ∫
                [f(x) + g(x)] dx = f(x) dx + g(x) dx

Some are pretty particular, like
                   ∫
                           1
                       √         dx = arcsec x + C.
                      x x2 − 1
What are we supposed to do with that?



                                                                               .   .   .     .       .      .

V63.0121.006/016, Calculus I (NYU)   Section 5.5 Integration by Substitution               April 27, 2010       7 / 38
No straightforward system of antidifferentiation



So far we don’t have any way to find
                           ∫
                                  2x
                              √         dx
                                 x2 + 1
or                                            ∫
                                                   tan x dx.




                                                                               .   .   .     .       .      .

V63.0121.006/016, Calculus I (NYU)   Section 5.5 Integration by Substitution               April 27, 2010       8 / 38
No straightforward system of antidifferentiation



So far we don’t have any way to find
                           ∫
                                  2x
                              √         dx
                                 x2 + 1
or                                            ∫
                                                   tan x dx.

Luckily, we can be smart and use the “anti” version of one of the most
important rules of differentiation: the chain rule.




                                                                               .   .   .     .       .      .

V63.0121.006/016, Calculus I (NYU)   Section 5.5 Integration by Substitution               April 27, 2010       8 / 38
Outline



Last Time: The Fundamental Theorem(s) of Calculus


Substitution for Indefinite Integrals
  Theory
  Examples


Substitution for Definite Integrals
  Theory
  Examples




                                                                               .   .   .     .       .      .

V63.0121.006/016, Calculus I (NYU)   Section 5.5 Integration by Substitution               April 27, 2010       9 / 38
Substitution for Indefinite Integrals



Example
Find                                       ∫
                                                  x
                                                √       dx.
                                                 x2 + 1




                                                                               .   .   .      .       .     .

V63.0121.006/016, Calculus I (NYU)   Section 5.5 Integration by Substitution               April 27, 2010   10 / 38
Substitution for Indefinite Integrals



Example
Find                                       ∫
                                                  x
                                                √       dx.
                                                 x2 + 1


Solution
Stare at this long enough and you notice the the integrand is the
                            √
derivative of the expression 1 + x2 .




                                                                               .   .   .      .       .     .

V63.0121.006/016, Calculus I (NYU)   Section 5.5 Integration by Substitution               April 27, 2010   10 / 38
Say what?


Solution (More slowly, now)
Let g(x) = x2 + 1.




                                                                               .   .   .      .       .     .

V63.0121.006/016, Calculus I (NYU)   Section 5.5 Integration by Substitution               April 27, 2010   11 / 38
Say what?


Solution (More slowly, now)
Let g(x) = x2 + 1. Then g′ (x) = 2x and so

                            d√         1               x
                               g(x) = √     g′ (x) = √
                            dx       2 g(x)           x2 + 1




                                                                               .   .   .      .       .     .

V63.0121.006/016, Calculus I (NYU)   Section 5.5 Integration by Substitution               April 27, 2010   11 / 38
Say what?


Solution (More slowly, now)
Let g(x) = x2 + 1. Then g′ (x) = 2x and so

                            d√         1               x
                               g(x) = √     g′ (x) = √
                            dx       2 g(x)           x2 + 1

Thus
                     ∫                         ∫ ()
                                x         d√
                          √       dx =        g(x) dx
                           x2 + 1        dx
                                       √          √
                                     = g(x) + C = 1 + x2 + C.



                                                                               .   .   .      .       .     .

V63.0121.006/016, Calculus I (NYU)   Section 5.5 Integration by Substitution               April 27, 2010   11 / 38
Leibnizian notation FTW


Solution (Same technique, new notation)
Let u = x2 + 1.




                                                                               .   .   .      .       .     .

V63.0121.006/016, Calculus I (NYU)   Section 5.5 Integration by Substitution               April 27, 2010   12 / 38
Leibnizian notation FTW


Solution (Same technique, new notation)
                                   √        √
Let u = x2 + 1. Then du = 2x dx and 1 + x2 = u.




                                                                               .   .   .      .       .     .

V63.0121.006/016, Calculus I (NYU)   Section 5.5 Integration by Substitution               April 27, 2010   12 / 38
Leibnizian notation FTW


Solution (Same technique, new notation)
                                   √        √
Let u = x2 + 1. Then du = 2x dx and 1 + x2 = u. So the integrand
becomes completely transformed into
                          ∫                         ∫     1            ∫
                               √
                                     x dx                 2 du
                                                          √
                                                                             1
                                                                             √ du
                                                =                 =
                                     x2   +1                 u              2 u




                                                                                    .   .   .      .       .     .

V63.0121.006/016, Calculus I (NYU)        Section 5.5 Integration by Substitution               April 27, 2010   12 / 38
Leibnizian notation FTW


Solution (Same technique, new notation)
                                   √        √
Let u = x2 + 1. Then du = 2x dx and 1 + x2 = u. So the integrand
becomes completely transformed into
                          ∫                         ∫     1            ∫
                               √
                                     x dx                 2 du
                                                          √
                                                                             1
                                                                             √ du
                                                =                 =
                                     x2   +1                 u              2 u
                                                    ∫
                                                         1 −1/2
                                                =        2u     du




                                                                                    .   .   .      .       .     .

V63.0121.006/016, Calculus I (NYU)        Section 5.5 Integration by Substitution               April 27, 2010   12 / 38
Leibnizian notation FTW


Solution (Same technique, new notation)
                                   √        √
Let u = x2 + 1. Then du = 2x dx and 1 + x2 = u. So the integrand
becomes completely transformed into
                          ∫                         ∫     1            ∫
                               √
                                     x dx                 2 du
                                                          √
                                                                             1
                                                                             √ du
                                                =                 =
                                     x2   +1                 u              2 u
                                                    ∫
                                                         1 −1/2
                                                =        2u     du
                                                    √       √
                                                =    u + C = 1 + x2 + C.




                                                                                    .   .   .      .       .     .

V63.0121.006/016, Calculus I (NYU)        Section 5.5 Integration by Substitution               April 27, 2010   12 / 38
Useful but unsavory variation
Solution (Same technique, new notation, more idiot-proof)
                                   √        √
Let u = x2 + 1. Then du = 2x dx and 1 + x2 = u. “Solve for dx:”
                                                           du
                                                dx =
                                                           2x
So the integrand becomes completely transformed into
              ∫              ∫             ∫
                     x          x du           1
                 √      dx =   √ ·       =    √ du
                   x2+1          u    2x     2 u
                             ∫
                               1 −1/2
                           =   2u      du
                             √          √
                           = u + C = 1 + x2 + C.


Mathematicians have serious issues with mixing the x and u like this.
However, I can’t deny that it works.                                           .   .   .      .       .     .

V63.0121.006/016, Calculus I (NYU)   Section 5.5 Integration by Substitution               April 27, 2010   13 / 38
Theorem of the Day

Theorem (The Substitution Rule)
If u = g(x) is a differentiable function whose range is an interval I and f
is continuous on I, then
                        ∫                   ∫
                                   ′
                           f(g(x))g (x) dx = f(u) du

That is, if F is an antiderivative for f, then
                         ∫
                            f(g(x))g′ (x) dx = F(g(x))

In Leibniz notation:
                                     ∫                           ∫
                                               du
                                           f(u) dx =                  f(u) du
                                               dx

                                                                                   .   .   .      .       .     .

V63.0121.006/016, Calculus I (NYU)       Section 5.5 Integration by Substitution               April 27, 2010   14 / 38
A polynomial example


Example
                                                              ∫
                                       2
Use the substitution u = x + 3 to find                            (x2 + 3)3 4x dx.




                                                                               .   .   .      .       .     .

V63.0121.006/016, Calculus I (NYU)   Section 5.5 Integration by Substitution               April 27, 2010   15 / 38
A polynomial example


Example
                                                              ∫
                                       2
Use the substitution u = x + 3 to find                            (x2 + 3)3 4x dx.


Solution
If u = x2 + 3, then du = 2x dx, and 4x dx = 2 du. So
                 ∫                 ∫             ∫
                   (x + 3) 4x dx = u 2du = 2 u3 du
                     2    3            3


                                                      1 4 1 2
                                                  =     u = (x + 3)4
                                                      2    2



                                                                               .   .   .      .       .     .

V63.0121.006/016, Calculus I (NYU)   Section 5.5 Integration by Substitution               April 27, 2010   15 / 38
A polynomial example, by brute force


Compare this to multiplying it out:
      ∫                    ∫ (                      )
           2     3
        (x + 3) 4x dx =         x6 + 9x4 + 27x2 + 27 4x dx
                           ∫ (                           )
                         =      4x7 + 36x5 + 108x3 + 108x dx
                                         1 8
                                     =     x + 6x6 + 27x4 + 54x2
                                         2




                                                                               .   .   .      .       .     .

V63.0121.006/016, Calculus I (NYU)   Section 5.5 Integration by Substitution               April 27, 2010   16 / 38
A polynomial example, by brute force


Compare this to multiplying it out:
      ∫                    ∫ (                      )
           2     3
        (x + 3) 4x dx =         x6 + 9x4 + 27x2 + 27 4x dx
                           ∫ (                           )
                         =      4x7 + 36x5 + 108x3 + 108x dx
                                         1 8
                                     =     x + 6x6 + 27x4 + 54x2
                                         2
Which would you rather do?




                                                                               .   .   .      .       .     .

V63.0121.006/016, Calculus I (NYU)   Section 5.5 Integration by Substitution               April 27, 2010   16 / 38
A polynomial example, by brute force


Compare this to multiplying it out:
      ∫                    ∫ (                      )
           2     3
        (x + 3) 4x dx =         x6 + 9x4 + 27x2 + 27 4x dx
                           ∫ (                           )
                         =      4x7 + 36x5 + 108x3 + 108x dx
                                         1 8
                                     =     x + 6x6 + 27x4 + 54x2
                                         2
Which would you rather do?
       It’s a wash for low powers
       But for higher powers, it’s much easier to do substitution.



                                                                               .   .   .      .       .     .

V63.0121.006/016, Calculus I (NYU)   Section 5.5 Integration by Substitution               April 27, 2010   16 / 38
Compare

We have the substitution method, which, when multiplied out, gives
    ∫
                        1
      (x2 + 3)3 4x dx = (x2 + 3)4
                        2
                        1( 8                             )
                      =    x + 12x6 + 54x4 + 108x2 + 81
                        2
                        1                          81
                      = x8 + 6x6 + 27x4 + 54x2 +
                        2                           2

and the brute force method
     ∫
                        1
       (x2 + 3)3 4x dx = x8 + 6x6 + 27x4 + 54x2
                        2
Is this a problem?

                                                                               .   .   .      .       .     .

V63.0121.006/016, Calculus I (NYU)   Section 5.5 Integration by Substitution               April 27, 2010   17 / 38
Compare

We have the substitution method, which, when multiplied out, gives
    ∫
                        1
      (x2 + 3)3 4x dx = (x2 + 3)4 + C
                        2
                        1( 8                             )
                      =    x + 12x6 + 54x4 + 108x2 + 81 + C
                        2
                        1                          81
                      = x8 + 6x6 + 27x4 + 54x2 +      +C
                        2                           2

and the brute force method
     ∫
                        1
       (x2 + 3)3 4x dx = x8 + 6x6 + 27x4 + 54x2 + C
                        2
Is this a problem? No, that’s what +C means!

                                                                               .   .   .      .       .     .

V63.0121.006/016, Calculus I (NYU)   Section 5.5 Integration by Substitution               April 27, 2010   17 / 38
A slick example


Example
    ∫
Find tan x dx.




                                                                               .   .   .      .       .     .

V63.0121.006/016, Calculus I (NYU)   Section 5.5 Integration by Substitution               April 27, 2010   18 / 38
A slick example


Example
    ∫
                              sin x
Find tan x dx. (Hint: tan x =       )
                              cos x




                                                                               .   .   .      .       .     .

V63.0121.006/016, Calculus I (NYU)   Section 5.5 Integration by Substitution               April 27, 2010   18 / 38
A slick example


Example
    ∫
                              sin x
Find tan x dx. (Hint: tan x =       )
                              cos x

Solution
Let u = cos x. Then du = − sin x dx.




                                                                               .   .   .      .       .     .

V63.0121.006/016, Calculus I (NYU)   Section 5.5 Integration by Substitution               April 27, 2010   18 / 38
A slick example


Example
    ∫
                              sin x
Find tan x dx. (Hint: tan x =       )
                              cos x

Solution
Let u = cos x. Then du = − sin x dx. So
             ∫             ∫              ∫
                             sin x          1
                tan x dx =         dx = −     du
                             cos x          u




                                                                               .   .   .      .       .     .

V63.0121.006/016, Calculus I (NYU)   Section 5.5 Integration by Substitution               April 27, 2010   18 / 38
A slick example


Example
    ∫
                              sin x
Find tan x dx. (Hint: tan x =       )
                              cos x

Solution
Let u = cos x. Then du = − sin x dx. So
             ∫             ∫                ∫
                              sin x           1
                tan x dx =           dx = −     du
                              cos x           u
                         = − ln |u| + C




                                                                               .   .   .      .       .     .

V63.0121.006/016, Calculus I (NYU)   Section 5.5 Integration by Substitution               April 27, 2010   18 / 38
A slick example


Example
    ∫
                              sin x
Find tan x dx. (Hint: tan x =       )
                              cos x

Solution
Let u = cos x. Then du = − sin x dx. So
             ∫             ∫                ∫
                              sin x           1
                tan x dx =           dx = −     du
                              cos x           u
                         = − ln |u| + C
                                     = − ln | cos x| + C = ln | sec x| + C



                                                                                .   .   .      .       .     .

V63.0121.006/016, Calculus I (NYU)    Section 5.5 Integration by Substitution               April 27, 2010   18 / 38
Can you do it another way?

Example
    ∫
                              sin x
Find tan x dx. (Hint: tan x =       )
                              cos x




                                                                               .   .   .      .       .     .

V63.0121.006/016, Calculus I (NYU)   Section 5.5 Integration by Substitution               April 27, 2010   19 / 38
Can you do it another way?

Example
    ∫
                              sin x
Find tan x dx. (Hint: tan x =       )
                              cos x

Solution
                                                                                    du
Let u = sin x. Then du = cos x dx and so dx =                                            .
                                                                                   cos x




                                                                               .       .     .      .       .     .

V63.0121.006/016, Calculus I (NYU)   Section 5.5 Integration by Substitution                     April 27, 2010   19 / 38
Can you do it another way?

Example
    ∫
                              sin x
Find tan x dx. (Hint: tan x =       )
                              cos x

Solution
                                                                                          du
Let u = sin x. Then du = cos x dx and so dx =                                                  .
                                                                                         cos x
                ∫                      ∫            ∫
                                         sin x           u     du
                     tan x dx =                dx =
                                         cos x         cos x cos x
                                       ∫          ∫                ∫
                                          u du          u du          u du
                                     =          =               =
                                         cos2 x              2
                                                     1 − sin x       1 − u2

At this point, although it’s possible to proceed, we should probably
back up and see if the other way works quicker (it does).
                                                                                     .       .     .      .       .     .

V63.0121.006/016, Calculus I (NYU)         Section 5.5 Integration by Substitution                     April 27, 2010   19 / 38
For those who really must know all

Solution (Continued, with algebra help)

             ∫                       ∫         ∫    (                )
                                      u du        1     1        1
                 tan x dx =                 =                −         du
                                     1 − u2       2 1−u 1+u
                                    1             1
                               = − ln |1 − u| − ln |1 + u| + C
                                    2             2
                                            1                       1
                               = ln √                 + C = ln √         +C
                                       (1 − u)(1 + u)             1 − u2
                                       1
                               = ln         + C = ln |sec x| + C
                                    |cos x|


There are other ways to do it, too.
                                                                                   .   .   .      .       .     .

V63.0121.006/016, Calculus I (NYU)       Section 5.5 Integration by Substitution               April 27, 2010   20 / 38
Outline



Last Time: The Fundamental Theorem(s) of Calculus


Substitution for Indefinite Integrals
  Theory
  Examples


Substitution for Definite Integrals
  Theory
  Examples




                                                                               .   .   .      .       .     .

V63.0121.006/016, Calculus I (NYU)   Section 5.5 Integration by Substitution               April 27, 2010   21 / 38
Substitution for Definite Integrals


Theorem (The Substitution Rule for Definite Integrals)
If g′ is continuous and f is continuous on the range of u = g(x), then
                              ∫      b                               ∫   g(b)
                                                    ′
                                         f(g(x))g (x) dx =                      f(u) du.
                                a                                      g(a)




                                                                                     .   .   .      .       .     .

V63.0121.006/016, Calculus I (NYU)         Section 5.5 Integration by Substitution               April 27, 2010   22 / 38
Substitution for Definite Integrals


Theorem (The Substitution Rule for Definite Integrals)
If g′ is continuous and f is continuous on the range of u = g(x), then
                              ∫      b                               ∫   g(b)
                                                    ′
                                         f(g(x))g (x) dx =                      f(u) du.
                                a                                      g(a)



Why the change in the limits?
       The integral on the left happens in “x-land”
       The integral on the right happens in “u-land”, so the limits need to
       be u-values
       To get from x to u, apply g


                                                                                     .   .   .      .       .     .

V63.0121.006/016, Calculus I (NYU)         Section 5.5 Integration by Substitution               April 27, 2010   22 / 38
Example
                ∫       π
Compute                     cos2 x sin x dx.
                    0




                                                                                  .   .   .      .       .     .

V63.0121.006/016, Calculus I (NYU)      Section 5.5 Integration by Substitution               April 27, 2010   23 / 38
Example
                ∫       π
Compute                     cos2 x sin x dx.
                    0


Solution (Slow Way)
                                                            ∫
First compute the indefinite integral                            cos2 x sin x dx and then
evaluate.




                                                                                  .   .   .      .       .     .

V63.0121.006/016, Calculus I (NYU)      Section 5.5 Integration by Substitution               April 27, 2010   23 / 38
Example
                 ∫       π
Compute                      cos2 x sin x dx.
                     0


Solution (Slow Way)
                                                             ∫
First compute the indefinite integral                             cos2 x sin x dx and then
evaluate. Let u = cos x. Then du = − sin x dx and
            ∫                    ∫
               cos x sin x dx = − u2 du
                  2


                                                = − 1 u3 + C = − 3 cos3 x + C.
                                                    3
                                                                 1


Therefore
      ∫ π
                                   1                             π
                                                                            1(           ) 2
                 cos x sin x dx = − cos3 x
                             2
                                                                     =−        (−1)3 − 13 = .
             0                     3                             0          3              3

                                                                                   .   .   .      .       .     .

V63.0121.006/016, Calculus I (NYU)       Section 5.5 Integration by Substitution               April 27, 2010   23 / 38
Definite-ly Quicker

Solution (Fast Way)
Do both the substitution and the evaluation at the same time.




                                                                               .   .   .      .       .     .

V63.0121.006/016, Calculus I (NYU)   Section 5.5 Integration by Substitution               April 27, 2010   24 / 38
Definite-ly Quicker

Solution (Fast Way)
Do both the substitution and the evaluation at the same time. Let
u = cos x. Then du = − sin x dx, u(0) = 1 and u(π) = −1.




                                                                               .   .   .      .       .     .

V63.0121.006/016, Calculus I (NYU)   Section 5.5 Integration by Substitution               April 27, 2010   24 / 38
Definite-ly Quicker

Solution (Fast Way)
Do both the substitution and the evaluation at the same time. Let
u = cos x. Then du = − sin x dx, u(0) = 1 and u(π) = −1. So
                  ∫     π                        ∫     −1                      ∫   1
                               2
                            cos x sin x dx =                −u du =
                                                                 2
                                                                                        u2 du
                    0                              1                               −1
                                                  1 3       1
                                                                     1(         ) 2
                                             =      u            =      1 − (−1) =
                                                  3         −1       3             3




                                                                               .        .   .      .       .     .

V63.0121.006/016, Calculus I (NYU)   Section 5.5 Integration by Substitution                    April 27, 2010   24 / 38
Definite-ly Quicker

Solution (Fast Way)
Do both the substitution and the evaluation at the same time. Let
u = cos x. Then du = − sin x dx, u(0) = 1 and u(π) = −1. So
                  ∫     π                        ∫     −1                      ∫   1
                               2
                            cos x sin x dx =                −u du =
                                                                 2
                                                                                        u2 du
                    0                              1                               −1
                                                  1 3       1
                                                                     1(         ) 2
                                             =      u            =      1 − (−1) =
                                                  3         −1       3             3


       The advantage to the “fast way” is that you completely transform
       the integral into something simpler and don’t have to go back to
       the original variable (x).
       But the slow way is just as reliable.
                                                                               .        .   .      .       .     .

V63.0121.006/016, Calculus I (NYU)   Section 5.5 Integration by Substitution                    April 27, 2010   24 / 38
An exponential example
Example
     ∫ ln √8    √
             2x
Find    √ e       e2x + 1 dx
          ln   3




                                                                               .   .   .      .       .     .

V63.0121.006/016, Calculus I (NYU)   Section 5.5 Integration by Substitution               April 27, 2010   25 / 38
An exponential example
Example
     ∫ ln √8    √
             2x
Find    √ e       e2x + 1 dx
          ln   3


Solution
Let u = e2x , so du = 2e2x dx. We have
                        ∫        √                                               ∫
                            ln       8            √                  1                  8√
                                             2x
                            √            e            e2x   + 1 dx =                            u + 1 du
                          ln 3                                       2              3




                                                                                            .      .   .      .       .     .

V63.0121.006/016, Calculus I (NYU)                Section 5.5 Integration by Substitution                  April 27, 2010   25 / 38
About those limits




Since
                                                    √               √    2
                                         e2(ln       3)
                                                           = eln        3
                                                                             = eln 3 = 3

we have                          √
                        ∫   ln       8            √                              ∫      8√
                                             2x                      1
                            √            e            e2x   + 1 dx =                            u + 1 du
                          ln 3                                       2              3




                                                                                            .      .   .      .       .     .

V63.0121.006/016, Calculus I (NYU)                Section 5.5 Integration by Substitution                  April 27, 2010   26 / 38
An exponential example
Example
     ∫ ln √8    √
             2x
Find    √ e       e2x + 1 dx
          ln   3


Solution
Let u = e2x , so du = 2e2x dx. We have
                           ∫        √                                               ∫
                               ln       8            √                  1                  8√
                                                2x
                             √              e            e2x   + 1 dx =                            u + 1 du
                           ln 3                                         2              3

Now let y = u + 1, dy = du. So
                       ∫   8√                                    ∫     9√                      ∫       9
                   1                       1                                      1
                                u + 1 du =                                 y dy =                          y1/2 dy
                   2   3                   2                       4              2                4
                                                                                9
                                                          1 2                        1           19
                                                         = · y3/2                   = (27 − 8) =
                                                          2 3                   4    3         .  3        .    .       .       .     .

V63.0121.006/016, Calculus I (NYU)                   Section 5.5 Integration by Substitution                         April 27, 2010   27 / 38
About those fractional powers



We have

                                      93/2 = (91/2 )3 = 33 = 27
                                      43/2 = (41/2 )3 = 23 = 8

so                      ∫     9                                 9
                    1                         1 2 3/2                   1            19
                                  y1/2 dy =    · y                  =     (27 − 8) =
                    2     4                   2 3               4       3             3




                                                                                  .   .   .      .       .     .

V63.0121.006/016, Calculus I (NYU)      Section 5.5 Integration by Substitution               April 27, 2010   28 / 38
An exponential example
Example
     ∫ ln √8    √
             2x
Find    √ e       e2x + 1 dx
          ln   3


Solution
Let u = e2x , so du = 2e2x dx. We have
                           ∫        √                                               ∫
                               ln       8            √                  1                  8√
                                                2x
                             √              e            e2x   + 1 dx =                            u + 1 du
                           ln 3                                         2              3

Now let y = u + 1, dy = du. So
                       ∫   8√                                    ∫     9√                      ∫       9
                   1                       1                                      1
                                u + 1 du =                                 y dy =                          y1/2 dy
                   2   3                   2                       4              2                4
                                                                                9
                                                          1 2                        1           19
                                                         = · y3/2                   = (27 − 8) =
                                                          2 3                   4    3         .  3        .    .       .       .     .

V63.0121.006/016, Calculus I (NYU)                   Section 5.5 Integration by Substitution                         April 27, 2010   29 / 38
Another way to skin that cat

Example
     ∫ ln √8    √
Find    √    e2x e2x + 1 dx
          ln   3


Solution
Let u = e2x + 1,




                                                                               .   .   .      .       .     .

V63.0121.006/016, Calculus I (NYU)   Section 5.5 Integration by Substitution               April 27, 2010   30 / 38
Another way to skin that cat

Example
     ∫ ln √8    √
Find    √    e2x e2x + 1 dx
          ln   3


Solution
Let u = e2x + 1,so that du = 2e2x dx.




                                                                               .   .   .      .       .     .

V63.0121.006/016, Calculus I (NYU)   Section 5.5 Integration by Substitution               April 27, 2010   30 / 38
Another way to skin that cat

Example
     ∫ ln √8    √
Find    √    e2x e2x + 1 dx
          ln   3


Solution
Let u = e2x + 1,so that du = 2e2x dx. Then
                        ∫        √                                                 ∫
                            ln         8            √                  1                 9√
                                               2x
                               √           e            e2x   + 1 dx =                            u du
                          ln       3                                   2             4




                                                                                              .      .   .      .       .     .

V63.0121.006/016, Calculus I (NYU)                  Section 5.5 Integration by Substitution                  April 27, 2010   30 / 38
Another way to skin that cat

Example
     ∫ ln √8    √
Find    √    e2x e2x + 1 dx
          ln   3


Solution
Let u = e2x + 1,so that du = 2e2x dx. Then
                        ∫        √                                                 ∫
                            ln         8            √                  1                 9√
                                               2x
                               √           e            e2x   + 1 dx =                            u du
                          ln       3                                   2             4
                                                                                              9
                                                                                1 3/2
                                                                          =       u
                                                                                3             4




                                                                                              .      .   .      .       .     .

V63.0121.006/016, Calculus I (NYU)                  Section 5.5 Integration by Substitution                  April 27, 2010   30 / 38
Another way to skin that cat

Example
     ∫ ln √8    √
Find    √    e2x e2x + 1 dx
          ln   3


Solution
Let u = e2x + 1,so that du = 2e2x dx. Then
                        ∫        √                                                 ∫
                            ln         8            √                  1                 9√
                                               2x
                               √           e            e2x   + 1 dx =                            u du
                          ln       3                                   2             4
                                                                           1 3/2 9
                                                                          =  u
                                                                           3     4
                                                                           1           19
                                                                          = (27 − 8) =
                                                                           3           3

                                                                                              .      .   .      .       .     .

V63.0121.006/016, Calculus I (NYU)                  Section 5.5 Integration by Substitution                  April 27, 2010   30 / 38
A third skinned cat

Example
     ∫ ln √8    √
             2x
Find    √ e       e2x + 1 dx
          ln   3


Solution
       √
Let u = e2x + 1, so that

                              u2 = e2x + 1




                                                                               .   .   .      .       .     .

V63.0121.006/016, Calculus I (NYU)   Section 5.5 Integration by Substitution               April 27, 2010   31 / 38
A third skinned cat

Example
     ∫ ln √8    √
             2x
Find    √ e       e2x + 1 dx
          ln   3


Solution
       √
Let u = e2x + 1, so that

                              u2 = e2x + 1 =⇒ 2u du = 2e2x dx




                                                                               .   .   .      .       .     .

V63.0121.006/016, Calculus I (NYU)   Section 5.5 Integration by Substitution               April 27, 2010   31 / 38
A third skinned cat

Example
     ∫ ln √8    √
             2x
Find    √ e       e2x + 1 dx
          ln   3


Solution
       √
Let u = e2x + 1, so that

                              u2 = e2x + 1 =⇒ 2u du = 2e2x dx

Thus                                  √
                             ∫   ln       8       ∫     3                           3
                                                                      1                         19
                                 √            =             u · u du = u3                   =
                               ln 3                 2                 3             2           3


                                                                                        .       .    .      .       .     .

V63.0121.006/016, Calculus I (NYU)            Section 5.5 Integration by Substitution                    April 27, 2010   31 / 38
A Trigonometric Example



Example
Find                             ∫              ( )      ( )
                                     3π/2
                                                 θ5     2 θ
                                            cot     sec      dθ.
                                     π           6        6




                                                                                   .   .   .      .       .     .

V63.0121.006/016, Calculus I (NYU)       Section 5.5 Integration by Substitution               April 27, 2010   32 / 38
A Trigonometric Example



Example
Find                             ∫              ( )      ( )
                                     3π/2
                                                 θ5     2 θ
                                            cot     sec      dθ.
                                     π           6        6

Before we dive in, think about:
       What “easy” substitutions might help?
       Which of the trig functions suggests a substitution?




                                                                                   .   .   .      .       .     .

V63.0121.006/016, Calculus I (NYU)       Section 5.5 Integration by Substitution               April 27, 2010   32 / 38
Solution
        θ             1
Let φ = . Then dφ = dθ.
        6             6
       ∫ 3π/2      ( )      ( )        ∫ π/4
                  5 θ      2 θ
              cot      sec      dθ = 6       cot5 φ sec2 φ dφ
        π           6        6          π/6
                                       ∫ π/4
                                             sec2 φ dφ
                                   =6
                                        π/6    tan5 φ




                                                                               .   .   .      .       .     .

V63.0121.006/016, Calculus I (NYU)   Section 5.5 Integration by Substitution               April 27, 2010   33 / 38
Solution
        θ             1
Let φ = . Then dφ = dθ.
        6             6
       ∫ 3π/2      ( )      ( )        ∫ π/4
                  5 θ      2 θ
              cot      sec      dθ = 6       cot5 φ sec2 φ dφ
        π           6        6          π/6
                                       ∫ π/4
                                             sec2 φ dφ
                                   =6
                                        π/6    tan5 φ

Now let u = tan φ. So du = sec2 φ dφ, and
                ∫   π/4                      ∫   1
                          sec2 φ dφ                 −5
            6                       =6           √ u   du
                 π/6       tan5 φ              1/ 3
                                              (       )
                                                  1 −4 1                           3
                                      =6         − u                √
                                                                           =         [9 − 1] = 12.
                                                  4               1/ 3             2


                                                                               .     .   .      .       .     .

V63.0121.006/016, Calculus I (NYU)   Section 5.5 Integration by Substitution                 April 27, 2010   33 / 38
The limits explained

                                              √
                                  π  sin π/4   2/2
                               tan =         =√    =1
                                  4  cos π/4   2/2
                                  π  sin π/6  1/2    1
                               tan =         =√    =√
                                  6  cos π/6   3/2    3




                                                                               .   .   .      .       .     .

V63.0121.006/016, Calculus I (NYU)   Section 5.5 Integration by Substitution               April 27, 2010   34 / 38
The limits explained

                                              √
                                  π  sin π/4   2/2
                               tan =         =√    =1
                                  4  cos π/4   2/2
                                  π  sin π/6  1/2    1
                               tan =         =√    =√
                                  6  cos π/6   3/2    3

                  (      )                                                     √
                    1 −4             1
                                                  3 [ −4 ]1         3 [ −4 ]1/ 3
                 6 − u                 √
                                                =    −u       √ =      u
                    4                1/ 3         2         1/ 3    2       1

                                                  3 [                        ]
                                                =    (3−1/2 )−4 − (1−1/2 )−4
                                                  2
                                                  3             3
                                                = [32 − 12 ] = (9 − 1) = 12
                                                  2             2

                                                                                   .   .   .      .       .     .

V63.0121.006/016, Calculus I (NYU)       Section 5.5 Integration by Substitution               April 27, 2010   34 / 38
Graphs
                       ∫   3π/2       ( )      ( )                                     ∫   π/4
                                       θ      2 θ
                       .          cot5
                                          sec      dθ                                  .         6 cot5 φ sec2 φ dφ
                       π               6        6                                      π/6

                  y
                  .                                                                y
                                                                                   .




                   .                               .                . .
                                                                      θ                    . .            .
                                                                                                          φ
                                      3π         .
                                                 π                                         ππ
                                     .                                                     . .
                                       2                                                   64
The areas of these two regions are the same.

                                                                                   .        .     .      .       .     .

V63.0121.006/016, Calculus I (NYU)       Section 5.5 Integration by Substitution                      April 27, 2010   35 / 38
Graphs                 ∫                                                               ∫
                           π/4                                                             1
                                     5            2                                                −5
                       .         6 cot φ sec φ dφ                                      . √ 6u           du
                        π/6                                                            1/ 3

                  y
                  .                                                                y
                                                                                   .




                   .       . .           .
                                         φ                                                 . ..
                                                                                              u
                           ππ                               1 .1
                           . .                             .√
                           64                                3
                       The areas of these two regions are the same.

                                                                                   .       .   .        .     .     .

V63.0121.006/016, Calculus I (NYU)       Section 5.5 Integration by Substitution                   April 27, 2010   36 / 38
Final Thoughts




       Antidifferentiation is a “nonlinear” problem that needs practice,
       intuition, and perserverance
       Worksheet in recitation (also to be posted)
       The whole antidifferentiation story is in Chapter 6




                                                                               .   .   .      .       .     .

V63.0121.006/016, Calculus I (NYU)   Section 5.5 Integration by Substitution               April 27, 2010   37 / 38
Summary




If F is an antiderivative for f, then:
                                         ∫
                                             f(g(x))g′ (x) dx = F(g(x))

                 ∫     b                             ∫   g(b)
                                     ′
                           f(g(x))g (x) dx =                    f(u) du = F(g(b)) − F(g(a))
                   a                                   g(a)




                                                                                   .   .   .      .       .     .

V63.0121.006/016, Calculus I (NYU)       Section 5.5 Integration by Substitution               April 27, 2010   38 / 38

More Related Content

What's hot

Limit & continuity, B.Sc . 1 calculus , Unit - 1
Limit & continuity, B.Sc . 1 calculus , Unit - 1Limit & continuity, B.Sc . 1 calculus , Unit - 1
Limit & continuity, B.Sc . 1 calculus , Unit - 1
Shri Shankaracharya College, Bhilai,Junwani
 
Lesson 14: Derivatives of Logarithmic and Exponential Functions
Lesson 14: Derivatives of Logarithmic and Exponential FunctionsLesson 14: Derivatives of Logarithmic and Exponential Functions
Lesson 14: Derivatives of Logarithmic and Exponential FunctionsMatthew Leingang
 
Limits and continuity powerpoint
Limits and continuity powerpointLimits and continuity powerpoint
Limits and continuity powerpointcanalculus
 
Funktion suurin ja pienin arvo laskemalla
Funktion suurin ja pienin arvo laskemallaFunktion suurin ja pienin arvo laskemalla
Funktion suurin ja pienin arvo laskemallateemunmatikka
 
Section 5.4 logarithmic functions
Section 5.4 logarithmic functions Section 5.4 logarithmic functions
Section 5.4 logarithmic functions
Wong Hsiung
 
3.2 implicit equations and implicit differentiation
3.2 implicit equations and implicit differentiation3.2 implicit equations and implicit differentiation
3.2 implicit equations and implicit differentiationmath265
 
Composite functions
Composite functionsComposite functions
Composite functionstoni dimella
 
TD - travaux dirigé limite de fonction ( exercice ) SOUFIANE MERABTI
TD - travaux dirigé limite de fonction ( exercice ) SOUFIANE MERABTITD - travaux dirigé limite de fonction ( exercice ) SOUFIANE MERABTI
TD - travaux dirigé limite de fonction ( exercice ) SOUFIANE MERABTI
soufiane merabti
 
Exponential and logrithmic functions
Exponential and logrithmic functionsExponential and logrithmic functions
Exponential and logrithmic functionsMalikahmad105
 
Partial Differentiation & Application
Partial Differentiation & Application Partial Differentiation & Application
Partial Differentiation & Application
Yana Qlah
 
Lesson 10: The Chain Rule
Lesson 10: The Chain RuleLesson 10: The Chain Rule
Lesson 10: The Chain Rule
Matthew Leingang
 
4 5 inverse functions
4 5 inverse functions4 5 inverse functions
4 5 inverse functionshisema01
 
Transformations of functions
Transformations of functionsTransformations of functions
Transformations of functions
KyleJohnson54657
 
2 integration and the substitution methods x
2 integration and the substitution methods x2 integration and the substitution methods x
2 integration and the substitution methods x
math266
 
limits and continuity
limits and continuity limits and continuity
limits and continuity
imran khan
 
Lesson 26: The Fundamental Theorem of Calculus (slides)
Lesson 26: The Fundamental Theorem of Calculus (slides)Lesson 26: The Fundamental Theorem of Calculus (slides)
Lesson 26: The Fundamental Theorem of Calculus (slides)
Matthew Leingang
 
4.5 continuous functions and differentiable functions
4.5 continuous functions and differentiable functions4.5 continuous functions and differentiable functions
4.5 continuous functions and differentiable functionsmath265
 
Lesson 11: Limits and Continuity
Lesson 11: Limits and ContinuityLesson 11: Limits and Continuity
Lesson 11: Limits and Continuity
Matthew Leingang
 
Continuity Of Functions
Continuity Of FunctionsContinuity Of Functions
Continuity Of Functions
Yash Thakkar
 

What's hot (20)

Limit & continuity, B.Sc . 1 calculus , Unit - 1
Limit & continuity, B.Sc . 1 calculus , Unit - 1Limit & continuity, B.Sc . 1 calculus , Unit - 1
Limit & continuity, B.Sc . 1 calculus , Unit - 1
 
Lesson 14: Derivatives of Logarithmic and Exponential Functions
Lesson 14: Derivatives of Logarithmic and Exponential FunctionsLesson 14: Derivatives of Logarithmic and Exponential Functions
Lesson 14: Derivatives of Logarithmic and Exponential Functions
 
Limits and continuity powerpoint
Limits and continuity powerpointLimits and continuity powerpoint
Limits and continuity powerpoint
 
Funktion suurin ja pienin arvo laskemalla
Funktion suurin ja pienin arvo laskemallaFunktion suurin ja pienin arvo laskemalla
Funktion suurin ja pienin arvo laskemalla
 
Section 5.4 logarithmic functions
Section 5.4 logarithmic functions Section 5.4 logarithmic functions
Section 5.4 logarithmic functions
 
3.2 implicit equations and implicit differentiation
3.2 implicit equations and implicit differentiation3.2 implicit equations and implicit differentiation
3.2 implicit equations and implicit differentiation
 
Composite functions
Composite functionsComposite functions
Composite functions
 
TD - travaux dirigé limite de fonction ( exercice ) SOUFIANE MERABTI
TD - travaux dirigé limite de fonction ( exercice ) SOUFIANE MERABTITD - travaux dirigé limite de fonction ( exercice ) SOUFIANE MERABTI
TD - travaux dirigé limite de fonction ( exercice ) SOUFIANE MERABTI
 
Functions limits and continuity
Functions limits and continuityFunctions limits and continuity
Functions limits and continuity
 
Exponential and logrithmic functions
Exponential and logrithmic functionsExponential and logrithmic functions
Exponential and logrithmic functions
 
Partial Differentiation & Application
Partial Differentiation & Application Partial Differentiation & Application
Partial Differentiation & Application
 
Lesson 10: The Chain Rule
Lesson 10: The Chain RuleLesson 10: The Chain Rule
Lesson 10: The Chain Rule
 
4 5 inverse functions
4 5 inverse functions4 5 inverse functions
4 5 inverse functions
 
Transformations of functions
Transformations of functionsTransformations of functions
Transformations of functions
 
2 integration and the substitution methods x
2 integration and the substitution methods x2 integration and the substitution methods x
2 integration and the substitution methods x
 
limits and continuity
limits and continuity limits and continuity
limits and continuity
 
Lesson 26: The Fundamental Theorem of Calculus (slides)
Lesson 26: The Fundamental Theorem of Calculus (slides)Lesson 26: The Fundamental Theorem of Calculus (slides)
Lesson 26: The Fundamental Theorem of Calculus (slides)
 
4.5 continuous functions and differentiable functions
4.5 continuous functions and differentiable functions4.5 continuous functions and differentiable functions
4.5 continuous functions and differentiable functions
 
Lesson 11: Limits and Continuity
Lesson 11: Limits and ContinuityLesson 11: Limits and Continuity
Lesson 11: Limits and Continuity
 
Continuity Of Functions
Continuity Of FunctionsContinuity Of Functions
Continuity Of Functions
 

Viewers also liked

12X1 T05 05 integration with inverse trig (2011)
12X1 T05 05 integration with inverse trig (2011)12X1 T05 05 integration with inverse trig (2011)
12X1 T05 05 integration with inverse trig (2011)Nigel Simmons
 
12 x1 t06 01 integration using substitution (2012)
12 x1 t06 01 integration using substitution (2012)12 x1 t06 01 integration using substitution (2012)
12 x1 t06 01 integration using substitution (2012)Nigel Simmons
 
Math pdf [eDvArDo]
Math pdf [eDvArDo]Math pdf [eDvArDo]
Math pdf [eDvArDo]
e D v A r D o
 
Calc i complete
Calc i complete Calc i complete
Calc i complete
harry_3k
 
5.2 the substitution methods
5.2 the substitution methods5.2 the substitution methods
5.2 the substitution methodsmath265
 
Integral Calculus
Integral CalculusIntegral Calculus
Integral Calculusitutor
 
Calc ii complete_solutions
Calc ii complete_solutionsCalc ii complete_solutions
Calc ii complete_solutionsAlelign Hailu
 
Calc ii complete_practice
Calc ii complete_practiceCalc ii complete_practice
Calc ii complete_practiceAlelign Hailu
 
Taking R to the Limit (High Performance Computing in R), Part 1 -- Paralleliz...
Taking R to the Limit (High Performance Computing in R), Part 1 -- Paralleliz...Taking R to the Limit (High Performance Computing in R), Part 1 -- Paralleliz...
Taking R to the Limit (High Performance Computing in R), Part 1 -- Paralleliz...Ryan Rosario
 
Calc ii complete
Calc ii completeCalc ii complete
Calc ii complete
yalfindira
 

Viewers also liked (13)

12X1 T05 05 integration with inverse trig (2011)
12X1 T05 05 integration with inverse trig (2011)12X1 T05 05 integration with inverse trig (2011)
12X1 T05 05 integration with inverse trig (2011)
 
12 x1 t06 01 integration using substitution (2012)
12 x1 t06 01 integration using substitution (2012)12 x1 t06 01 integration using substitution (2012)
12 x1 t06 01 integration using substitution (2012)
 
Calc 5.7
Calc 5.7Calc 5.7
Calc 5.7
 
Integration
IntegrationIntegration
Integration
 
Math pdf [eDvArDo]
Math pdf [eDvArDo]Math pdf [eDvArDo]
Math pdf [eDvArDo]
 
Calc i complete
Calc i complete Calc i complete
Calc i complete
 
5.2 the substitution methods
5.2 the substitution methods5.2 the substitution methods
5.2 the substitution methods
 
Integral Calculus
Integral CalculusIntegral Calculus
Integral Calculus
 
Calc ii complete_solutions
Calc ii complete_solutionsCalc ii complete_solutions
Calc ii complete_solutions
 
Calc ii complete
Calc ii completeCalc ii complete
Calc ii complete
 
Calc ii complete_practice
Calc ii complete_practiceCalc ii complete_practice
Calc ii complete_practice
 
Taking R to the Limit (High Performance Computing in R), Part 1 -- Paralleliz...
Taking R to the Limit (High Performance Computing in R), Part 1 -- Paralleliz...Taking R to the Limit (High Performance Computing in R), Part 1 -- Paralleliz...
Taking R to the Limit (High Performance Computing in R), Part 1 -- Paralleliz...
 
Calc ii complete
Calc ii completeCalc ii complete
Calc ii complete
 

Similar to Lesson 26: Integration by Substitution (slides)

Lesson 28: Integration by Subsitution
Lesson 28: Integration by SubsitutionLesson 28: Integration by Subsitution
Lesson 28: Integration by SubsitutionMatthew Leingang
 
Lesson 28: Integration by Subsitution
Lesson 28: Integration by SubsitutionLesson 28: Integration by Subsitution
Lesson 28: Integration by SubsitutionMel Anthony Pepito
 
Lesson 25: The Fundamental Theorem of Calculus
Lesson 25: The Fundamental Theorem of CalculusLesson 25: The Fundamental Theorem of Calculus
Lesson 25: The Fundamental Theorem of CalculusMatthew Leingang
 
Lesson 27: Integration by Substitution (Section 041 handout)
Lesson 27: Integration by Substitution (Section 041 handout)Lesson 27: Integration by Substitution (Section 041 handout)
Lesson 27: Integration by Substitution (Section 041 handout)
Matthew Leingang
 
Lesson 26: The Fundamental Theorem of Calculus (Section 041 slides)
Lesson 26: The Fundamental Theorem of Calculus (Section 041 slides)Lesson 26: The Fundamental Theorem of Calculus (Section 041 slides)
Lesson 26: The Fundamental Theorem of Calculus (Section 041 slides)
Mel Anthony Pepito
 
Lesson 26: The Fundamental Theorem of Calculus (Section 041 slides)
Lesson 26: The Fundamental Theorem of Calculus (Section 041 slides)Lesson 26: The Fundamental Theorem of Calculus (Section 041 slides)
Lesson 26: The Fundamental Theorem of Calculus (Section 041 slides)
Matthew Leingang
 
Lesson 27: Integration by Substitution (Section 021 handout)
Lesson 27: Integration by Substitution (Section 021 handout)Lesson 27: Integration by Substitution (Section 021 handout)
Lesson 27: Integration by Substitution (Section 021 handout)
Matthew Leingang
 
Lesson 26: Integration by Substitution (handout)
Lesson 26: Integration by Substitution (handout)Lesson 26: Integration by Substitution (handout)
Lesson 26: Integration by Substitution (handout)Matthew Leingang
 
Lesson 21: Antiderivatives (slides)
Lesson 21: Antiderivatives (slides)Lesson 21: Antiderivatives (slides)
Lesson 21: Antiderivatives (slides)
Matthew Leingang
 
Lesson 26: The Fundamental Theorem of Calculus (Section 021 slides)
Lesson 26: The Fundamental Theorem of Calculus (Section 021 slides)Lesson 26: The Fundamental Theorem of Calculus (Section 021 slides)
Lesson 26: The Fundamental Theorem of Calculus (Section 021 slides)
Matthew Leingang
 
Lesson 26: The Fundamental Theorem of Calculus (Section 021 slides)
Lesson 26: The Fundamental Theorem of Calculus (Section 021 slides)Lesson 26: The Fundamental Theorem of Calculus (Section 021 slides)
Lesson 26: The Fundamental Theorem of Calculus (Section 021 slides)
Mel Anthony Pepito
 
Lesson 26: The Fundamental Theorem of Calculus (Section 041 handout)
Lesson 26: The Fundamental Theorem of Calculus (Section 041 handout)Lesson 26: The Fundamental Theorem of Calculus (Section 041 handout)
Lesson 26: The Fundamental Theorem of Calculus (Section 041 handout)
Matthew Leingang
 
Lesson 27: Integration by Substitution (Section 041 slides)
Lesson 27: Integration by Substitution (Section 041 slides)Lesson 27: Integration by Substitution (Section 041 slides)
Lesson 27: Integration by Substitution (Section 041 slides)
Matthew Leingang
 
Lesson 27: Integration by Substitution (Section 041 slides)
Lesson 27: Integration by Substitution (Section 041 slides)Lesson 27: Integration by Substitution (Section 041 slides)
Lesson 27: Integration by Substitution (Section 041 slides)
Mel Anthony Pepito
 
Lesson 21: Antiderivatives (notes)
Lesson 21: Antiderivatives (notes)Lesson 21: Antiderivatives (notes)
Lesson 21: Antiderivatives (notes)
Matthew Leingang
 
Lesson 23: Antiderivatives
Lesson 23: AntiderivativesLesson 23: Antiderivatives
Lesson 23: AntiderivativesMatthew Leingang
 
Lesson 23: Antiderivatives (Section 021 slides)
Lesson 23: Antiderivatives (Section 021 slides)Lesson 23: Antiderivatives (Section 021 slides)
Lesson 23: Antiderivatives (Section 021 slides)
Mel Anthony Pepito
 
Lesson 27: Integration by Substitution (handout)
Lesson 27: Integration by Substitution (handout)Lesson 27: Integration by Substitution (handout)
Lesson 27: Integration by Substitution (handout)
Matthew Leingang
 
Lesson 25: The Fundamental Theorem of Calculus (handout)
Lesson 25: The Fundamental Theorem of Calculus (handout)Lesson 25: The Fundamental Theorem of Calculus (handout)
Lesson 25: The Fundamental Theorem of Calculus (handout)Matthew Leingang
 

Similar to Lesson 26: Integration by Substitution (slides) (20)

Lesson 28: Integration by Subsitution
Lesson 28: Integration by SubsitutionLesson 28: Integration by Subsitution
Lesson 28: Integration by Subsitution
 
Lesson 28: Integration by Subsitution
Lesson 28: Integration by SubsitutionLesson 28: Integration by Subsitution
Lesson 28: Integration by Subsitution
 
Lesson 25: The Fundamental Theorem of Calculus
Lesson 25: The Fundamental Theorem of CalculusLesson 25: The Fundamental Theorem of Calculus
Lesson 25: The Fundamental Theorem of Calculus
 
Lesson 27: Integration by Substitution (Section 041 handout)
Lesson 27: Integration by Substitution (Section 041 handout)Lesson 27: Integration by Substitution (Section 041 handout)
Lesson 27: Integration by Substitution (Section 041 handout)
 
Lesson 26: The Fundamental Theorem of Calculus (Section 041 slides)
Lesson 26: The Fundamental Theorem of Calculus (Section 041 slides)Lesson 26: The Fundamental Theorem of Calculus (Section 041 slides)
Lesson 26: The Fundamental Theorem of Calculus (Section 041 slides)
 
Lesson 26: The Fundamental Theorem of Calculus (Section 041 slides)
Lesson 26: The Fundamental Theorem of Calculus (Section 041 slides)Lesson 26: The Fundamental Theorem of Calculus (Section 041 slides)
Lesson 26: The Fundamental Theorem of Calculus (Section 041 slides)
 
Lesson 27: Integration by Substitution (Section 021 handout)
Lesson 27: Integration by Substitution (Section 021 handout)Lesson 27: Integration by Substitution (Section 021 handout)
Lesson 27: Integration by Substitution (Section 021 handout)
 
Lesson 26: Integration by Substitution (handout)
Lesson 26: Integration by Substitution (handout)Lesson 26: Integration by Substitution (handout)
Lesson 26: Integration by Substitution (handout)
 
Lesson 21: Antiderivatives (slides)
Lesson 21: Antiderivatives (slides)Lesson 21: Antiderivatives (slides)
Lesson 21: Antiderivatives (slides)
 
Lesson 26: The Fundamental Theorem of Calculus (Section 021 slides)
Lesson 26: The Fundamental Theorem of Calculus (Section 021 slides)Lesson 26: The Fundamental Theorem of Calculus (Section 021 slides)
Lesson 26: The Fundamental Theorem of Calculus (Section 021 slides)
 
Lesson 26: The Fundamental Theorem of Calculus (Section 021 slides)
Lesson 26: The Fundamental Theorem of Calculus (Section 021 slides)Lesson 26: The Fundamental Theorem of Calculus (Section 021 slides)
Lesson 26: The Fundamental Theorem of Calculus (Section 021 slides)
 
Lesson 26: The Fundamental Theorem of Calculus (Section 041 handout)
Lesson 26: The Fundamental Theorem of Calculus (Section 041 handout)Lesson 26: The Fundamental Theorem of Calculus (Section 041 handout)
Lesson 26: The Fundamental Theorem of Calculus (Section 041 handout)
 
Lesson 27: Integration by Substitution (Section 041 slides)
Lesson 27: Integration by Substitution (Section 041 slides)Lesson 27: Integration by Substitution (Section 041 slides)
Lesson 27: Integration by Substitution (Section 041 slides)
 
Lesson 27: Integration by Substitution (Section 041 slides)
Lesson 27: Integration by Substitution (Section 041 slides)Lesson 27: Integration by Substitution (Section 041 slides)
Lesson 27: Integration by Substitution (Section 041 slides)
 
Lesson 21: Antiderivatives (notes)
Lesson 21: Antiderivatives (notes)Lesson 21: Antiderivatives (notes)
Lesson 21: Antiderivatives (notes)
 
Lesson 23: Antiderivatives
Lesson 23: AntiderivativesLesson 23: Antiderivatives
Lesson 23: Antiderivatives
 
Lesson 23: Antiderivatives
Lesson 23: AntiderivativesLesson 23: Antiderivatives
Lesson 23: Antiderivatives
 
Lesson 23: Antiderivatives (Section 021 slides)
Lesson 23: Antiderivatives (Section 021 slides)Lesson 23: Antiderivatives (Section 021 slides)
Lesson 23: Antiderivatives (Section 021 slides)
 
Lesson 27: Integration by Substitution (handout)
Lesson 27: Integration by Substitution (handout)Lesson 27: Integration by Substitution (handout)
Lesson 27: Integration by Substitution (handout)
 
Lesson 25: The Fundamental Theorem of Calculus (handout)
Lesson 25: The Fundamental Theorem of Calculus (handout)Lesson 25: The Fundamental Theorem of Calculus (handout)
Lesson 25: The Fundamental Theorem of Calculus (handout)
 

More from Matthew Leingang

Making Lesson Plans
Making Lesson PlansMaking Lesson Plans
Making Lesson Plans
Matthew Leingang
 
Streamlining assessment, feedback, and archival with auto-multiple-choice
Streamlining assessment, feedback, and archival with auto-multiple-choiceStreamlining assessment, feedback, and archival with auto-multiple-choice
Streamlining assessment, feedback, and archival with auto-multiple-choice
Matthew Leingang
 
Electronic Grading of Paper Assessments
Electronic Grading of Paper AssessmentsElectronic Grading of Paper Assessments
Electronic Grading of Paper Assessments
Matthew Leingang
 
Lesson 27: Integration by Substitution (slides)
Lesson 27: Integration by Substitution (slides)Lesson 27: Integration by Substitution (slides)
Lesson 27: Integration by Substitution (slides)
Matthew Leingang
 
Lesson 26: The Fundamental Theorem of Calculus (slides)
Lesson 26: The Fundamental Theorem of Calculus (slides)Lesson 26: The Fundamental Theorem of Calculus (slides)
Lesson 26: The Fundamental Theorem of Calculus (slides)
Matthew Leingang
 
Lesson 26: The Fundamental Theorem of Calculus (handout)
Lesson 26: The Fundamental Theorem of Calculus (handout)Lesson 26: The Fundamental Theorem of Calculus (handout)
Lesson 26: The Fundamental Theorem of Calculus (handout)
Matthew Leingang
 
Lesson 25: Evaluating Definite Integrals (slides)
Lesson 25: Evaluating Definite Integrals (slides)Lesson 25: Evaluating Definite Integrals (slides)
Lesson 25: Evaluating Definite Integrals (slides)
Matthew Leingang
 
Lesson 25: Evaluating Definite Integrals (handout)
Lesson 25: Evaluating Definite Integrals (handout)Lesson 25: Evaluating Definite Integrals (handout)
Lesson 25: Evaluating Definite Integrals (handout)
Matthew Leingang
 
Lesson 24: Areas and Distances, The Definite Integral (handout)
Lesson 24: Areas and Distances, The Definite Integral (handout)Lesson 24: Areas and Distances, The Definite Integral (handout)
Lesson 24: Areas and Distances, The Definite Integral (handout)
Matthew Leingang
 
Lesson 24: Areas and Distances, The Definite Integral (slides)
Lesson 24: Areas and Distances, The Definite Integral (slides)Lesson 24: Areas and Distances, The Definite Integral (slides)
Lesson 24: Areas and Distances, The Definite Integral (slides)
Matthew Leingang
 
Lesson 23: Antiderivatives (slides)
Lesson 23: Antiderivatives (slides)Lesson 23: Antiderivatives (slides)
Lesson 23: Antiderivatives (slides)
Matthew Leingang
 
Lesson 23: Antiderivatives (slides)
Lesson 23: Antiderivatives (slides)Lesson 23: Antiderivatives (slides)
Lesson 23: Antiderivatives (slides)
Matthew Leingang
 
Lesson 22: Optimization Problems (slides)
Lesson 22: Optimization Problems (slides)Lesson 22: Optimization Problems (slides)
Lesson 22: Optimization Problems (slides)
Matthew Leingang
 
Lesson 22: Optimization Problems (handout)
Lesson 22: Optimization Problems (handout)Lesson 22: Optimization Problems (handout)
Lesson 22: Optimization Problems (handout)
Matthew Leingang
 
Lesson 21: Curve Sketching (slides)
Lesson 21: Curve Sketching (slides)Lesson 21: Curve Sketching (slides)
Lesson 21: Curve Sketching (slides)
Matthew Leingang
 
Lesson 21: Curve Sketching (handout)
Lesson 21: Curve Sketching (handout)Lesson 21: Curve Sketching (handout)
Lesson 21: Curve Sketching (handout)
Matthew Leingang
 
Lesson 20: Derivatives and the Shapes of Curves (slides)
Lesson 20: Derivatives and the Shapes of Curves (slides)Lesson 20: Derivatives and the Shapes of Curves (slides)
Lesson 20: Derivatives and the Shapes of Curves (slides)
Matthew Leingang
 
Lesson 20: Derivatives and the Shapes of Curves (handout)
Lesson 20: Derivatives and the Shapes of Curves (handout)Lesson 20: Derivatives and the Shapes of Curves (handout)
Lesson 20: Derivatives and the Shapes of Curves (handout)
Matthew Leingang
 
Lesson 19: The Mean Value Theorem (slides)
Lesson 19: The Mean Value Theorem (slides)Lesson 19: The Mean Value Theorem (slides)
Lesson 19: The Mean Value Theorem (slides)
Matthew Leingang
 
Lesson 18: Maximum and Minimum Values (slides)
Lesson 18: Maximum and Minimum Values (slides)Lesson 18: Maximum and Minimum Values (slides)
Lesson 18: Maximum and Minimum Values (slides)
Matthew Leingang
 

More from Matthew Leingang (20)

Making Lesson Plans
Making Lesson PlansMaking Lesson Plans
Making Lesson Plans
 
Streamlining assessment, feedback, and archival with auto-multiple-choice
Streamlining assessment, feedback, and archival with auto-multiple-choiceStreamlining assessment, feedback, and archival with auto-multiple-choice
Streamlining assessment, feedback, and archival with auto-multiple-choice
 
Electronic Grading of Paper Assessments
Electronic Grading of Paper AssessmentsElectronic Grading of Paper Assessments
Electronic Grading of Paper Assessments
 
Lesson 27: Integration by Substitution (slides)
Lesson 27: Integration by Substitution (slides)Lesson 27: Integration by Substitution (slides)
Lesson 27: Integration by Substitution (slides)
 
Lesson 26: The Fundamental Theorem of Calculus (slides)
Lesson 26: The Fundamental Theorem of Calculus (slides)Lesson 26: The Fundamental Theorem of Calculus (slides)
Lesson 26: The Fundamental Theorem of Calculus (slides)
 
Lesson 26: The Fundamental Theorem of Calculus (handout)
Lesson 26: The Fundamental Theorem of Calculus (handout)Lesson 26: The Fundamental Theorem of Calculus (handout)
Lesson 26: The Fundamental Theorem of Calculus (handout)
 
Lesson 25: Evaluating Definite Integrals (slides)
Lesson 25: Evaluating Definite Integrals (slides)Lesson 25: Evaluating Definite Integrals (slides)
Lesson 25: Evaluating Definite Integrals (slides)
 
Lesson 25: Evaluating Definite Integrals (handout)
Lesson 25: Evaluating Definite Integrals (handout)Lesson 25: Evaluating Definite Integrals (handout)
Lesson 25: Evaluating Definite Integrals (handout)
 
Lesson 24: Areas and Distances, The Definite Integral (handout)
Lesson 24: Areas and Distances, The Definite Integral (handout)Lesson 24: Areas and Distances, The Definite Integral (handout)
Lesson 24: Areas and Distances, The Definite Integral (handout)
 
Lesson 24: Areas and Distances, The Definite Integral (slides)
Lesson 24: Areas and Distances, The Definite Integral (slides)Lesson 24: Areas and Distances, The Definite Integral (slides)
Lesson 24: Areas and Distances, The Definite Integral (slides)
 
Lesson 23: Antiderivatives (slides)
Lesson 23: Antiderivatives (slides)Lesson 23: Antiderivatives (slides)
Lesson 23: Antiderivatives (slides)
 
Lesson 23: Antiderivatives (slides)
Lesson 23: Antiderivatives (slides)Lesson 23: Antiderivatives (slides)
Lesson 23: Antiderivatives (slides)
 
Lesson 22: Optimization Problems (slides)
Lesson 22: Optimization Problems (slides)Lesson 22: Optimization Problems (slides)
Lesson 22: Optimization Problems (slides)
 
Lesson 22: Optimization Problems (handout)
Lesson 22: Optimization Problems (handout)Lesson 22: Optimization Problems (handout)
Lesson 22: Optimization Problems (handout)
 
Lesson 21: Curve Sketching (slides)
Lesson 21: Curve Sketching (slides)Lesson 21: Curve Sketching (slides)
Lesson 21: Curve Sketching (slides)
 
Lesson 21: Curve Sketching (handout)
Lesson 21: Curve Sketching (handout)Lesson 21: Curve Sketching (handout)
Lesson 21: Curve Sketching (handout)
 
Lesson 20: Derivatives and the Shapes of Curves (slides)
Lesson 20: Derivatives and the Shapes of Curves (slides)Lesson 20: Derivatives and the Shapes of Curves (slides)
Lesson 20: Derivatives and the Shapes of Curves (slides)
 
Lesson 20: Derivatives and the Shapes of Curves (handout)
Lesson 20: Derivatives and the Shapes of Curves (handout)Lesson 20: Derivatives and the Shapes of Curves (handout)
Lesson 20: Derivatives and the Shapes of Curves (handout)
 
Lesson 19: The Mean Value Theorem (slides)
Lesson 19: The Mean Value Theorem (slides)Lesson 19: The Mean Value Theorem (slides)
Lesson 19: The Mean Value Theorem (slides)
 
Lesson 18: Maximum and Minimum Values (slides)
Lesson 18: Maximum and Minimum Values (slides)Lesson 18: Maximum and Minimum Values (slides)
Lesson 18: Maximum and Minimum Values (slides)
 

Lesson 26: Integration by Substitution (slides)

  • 1. Section 5.5 Integration by Substitution V63.0121.006/016, Calculus I New York University April 27, 2010 Announcements April 29: Movie Day April 30: Quiz 5 on §§5.1–5.4 Monday, May 10, 12:00noon Final Exam: SILV 703 (Section 16) MEYR 121/122 (Section 6) . . . . . .
  • 2. Announcements April 29: Movie Day April 30: Quiz 5 on §§5.1–5.4 Monday, May 10, 12:00noon Final Exam: SILV 703 (Section 16) MEYR 121/122 (Section 6) . . . . . . V63.0121.006/016, Calculus I (NYU) Section 5.5 Integration by Substitution April 27, 2010 2 / 38
  • 3. Resurrection Policy If your final score beats your midterm score, we will add 10% to its weight, and subtract 10% from the midterm weight. . . . . . . . Image credit: Scott Beale / Laughing Squid V63.0121.006/016, Calculus I (NYU) Section 5.5 Integration by Substitution April 27, 2010 3 / 38
  • 4. Objectives Given an integral and a substitution, transform the integral into an equivalent one using a substitution Evaluate indefinite integrals using the method of substitution. Evaluate definite integrals using the method of substitution. . . . . . . V63.0121.006/016, Calculus I (NYU) Section 5.5 Integration by Substitution April 27, 2010 4 / 38
  • 5. Outline Last Time: The Fundamental Theorem(s) of Calculus Substitution for Indefinite Integrals Theory Examples Substitution for Definite Integrals Theory Examples . . . . . . V63.0121.006/016, Calculus I (NYU) Section 5.5 Integration by Substitution April 27, 2010 5 / 38
  • 6. Differentiation and Integration as reverse processes Theorem (The Fundamental Theorem of Calculus) 1. Let f be continuous on [a, b]. Then ∫ x d f(t) dt = f(x) dx a 2. Let f be continuous on [a, b] and f = F′ for some other function F. Then ∫ b f(x) dx = F(b) − F(a). a . . . . . . V63.0121.006/016, Calculus I (NYU) Section 5.5 Integration by Substitution April 27, 2010 6 / 38
  • 7. Techniques of antidifferentiation? So far we know only a few rules for antidifferentiation. Some are general, like ∫ ∫ ∫ [f(x) + g(x)] dx = f(x) dx + g(x) dx . . . . . . V63.0121.006/016, Calculus I (NYU) Section 5.5 Integration by Substitution April 27, 2010 7 / 38
  • 8. Techniques of antidifferentiation? So far we know only a few rules for antidifferentiation. Some are general, like ∫ ∫ ∫ [f(x) + g(x)] dx = f(x) dx + g(x) dx Some are pretty particular, like ∫ 1 √ dx = arcsec x + C. x x2 − 1 . . . . . . V63.0121.006/016, Calculus I (NYU) Section 5.5 Integration by Substitution April 27, 2010 7 / 38
  • 9. Techniques of antidifferentiation? So far we know only a few rules for antidifferentiation. Some are general, like ∫ ∫ ∫ [f(x) + g(x)] dx = f(x) dx + g(x) dx Some are pretty particular, like ∫ 1 √ dx = arcsec x + C. x x2 − 1 What are we supposed to do with that? . . . . . . V63.0121.006/016, Calculus I (NYU) Section 5.5 Integration by Substitution April 27, 2010 7 / 38
  • 10. No straightforward system of antidifferentiation So far we don’t have any way to find ∫ 2x √ dx x2 + 1 or ∫ tan x dx. . . . . . . V63.0121.006/016, Calculus I (NYU) Section 5.5 Integration by Substitution April 27, 2010 8 / 38
  • 11. No straightforward system of antidifferentiation So far we don’t have any way to find ∫ 2x √ dx x2 + 1 or ∫ tan x dx. Luckily, we can be smart and use the “anti” version of one of the most important rules of differentiation: the chain rule. . . . . . . V63.0121.006/016, Calculus I (NYU) Section 5.5 Integration by Substitution April 27, 2010 8 / 38
  • 12. Outline Last Time: The Fundamental Theorem(s) of Calculus Substitution for Indefinite Integrals Theory Examples Substitution for Definite Integrals Theory Examples . . . . . . V63.0121.006/016, Calculus I (NYU) Section 5.5 Integration by Substitution April 27, 2010 9 / 38
  • 13. Substitution for Indefinite Integrals Example Find ∫ x √ dx. x2 + 1 . . . . . . V63.0121.006/016, Calculus I (NYU) Section 5.5 Integration by Substitution April 27, 2010 10 / 38
  • 14. Substitution for Indefinite Integrals Example Find ∫ x √ dx. x2 + 1 Solution Stare at this long enough and you notice the the integrand is the √ derivative of the expression 1 + x2 . . . . . . . V63.0121.006/016, Calculus I (NYU) Section 5.5 Integration by Substitution April 27, 2010 10 / 38
  • 15. Say what? Solution (More slowly, now) Let g(x) = x2 + 1. . . . . . . V63.0121.006/016, Calculus I (NYU) Section 5.5 Integration by Substitution April 27, 2010 11 / 38
  • 16. Say what? Solution (More slowly, now) Let g(x) = x2 + 1. Then g′ (x) = 2x and so d√ 1 x g(x) = √ g′ (x) = √ dx 2 g(x) x2 + 1 . . . . . . V63.0121.006/016, Calculus I (NYU) Section 5.5 Integration by Substitution April 27, 2010 11 / 38
  • 17. Say what? Solution (More slowly, now) Let g(x) = x2 + 1. Then g′ (x) = 2x and so d√ 1 x g(x) = √ g′ (x) = √ dx 2 g(x) x2 + 1 Thus ∫ ∫ () x d√ √ dx = g(x) dx x2 + 1 dx √ √ = g(x) + C = 1 + x2 + C. . . . . . . V63.0121.006/016, Calculus I (NYU) Section 5.5 Integration by Substitution April 27, 2010 11 / 38
  • 18. Leibnizian notation FTW Solution (Same technique, new notation) Let u = x2 + 1. . . . . . . V63.0121.006/016, Calculus I (NYU) Section 5.5 Integration by Substitution April 27, 2010 12 / 38
  • 19. Leibnizian notation FTW Solution (Same technique, new notation) √ √ Let u = x2 + 1. Then du = 2x dx and 1 + x2 = u. . . . . . . V63.0121.006/016, Calculus I (NYU) Section 5.5 Integration by Substitution April 27, 2010 12 / 38
  • 20. Leibnizian notation FTW Solution (Same technique, new notation) √ √ Let u = x2 + 1. Then du = 2x dx and 1 + x2 = u. So the integrand becomes completely transformed into ∫ ∫ 1 ∫ √ x dx 2 du √ 1 √ du = = x2 +1 u 2 u . . . . . . V63.0121.006/016, Calculus I (NYU) Section 5.5 Integration by Substitution April 27, 2010 12 / 38
  • 21. Leibnizian notation FTW Solution (Same technique, new notation) √ √ Let u = x2 + 1. Then du = 2x dx and 1 + x2 = u. So the integrand becomes completely transformed into ∫ ∫ 1 ∫ √ x dx 2 du √ 1 √ du = = x2 +1 u 2 u ∫ 1 −1/2 = 2u du . . . . . . V63.0121.006/016, Calculus I (NYU) Section 5.5 Integration by Substitution April 27, 2010 12 / 38
  • 22. Leibnizian notation FTW Solution (Same technique, new notation) √ √ Let u = x2 + 1. Then du = 2x dx and 1 + x2 = u. So the integrand becomes completely transformed into ∫ ∫ 1 ∫ √ x dx 2 du √ 1 √ du = = x2 +1 u 2 u ∫ 1 −1/2 = 2u du √ √ = u + C = 1 + x2 + C. . . . . . . V63.0121.006/016, Calculus I (NYU) Section 5.5 Integration by Substitution April 27, 2010 12 / 38
  • 23. Useful but unsavory variation Solution (Same technique, new notation, more idiot-proof) √ √ Let u = x2 + 1. Then du = 2x dx and 1 + x2 = u. “Solve for dx:” du dx = 2x So the integrand becomes completely transformed into ∫ ∫ ∫ x x du 1 √ dx = √ · = √ du x2+1 u 2x 2 u ∫ 1 −1/2 = 2u du √ √ = u + C = 1 + x2 + C. Mathematicians have serious issues with mixing the x and u like this. However, I can’t deny that it works. . . . . . . V63.0121.006/016, Calculus I (NYU) Section 5.5 Integration by Substitution April 27, 2010 13 / 38
  • 24. Theorem of the Day Theorem (The Substitution Rule) If u = g(x) is a differentiable function whose range is an interval I and f is continuous on I, then ∫ ∫ ′ f(g(x))g (x) dx = f(u) du That is, if F is an antiderivative for f, then ∫ f(g(x))g′ (x) dx = F(g(x)) In Leibniz notation: ∫ ∫ du f(u) dx = f(u) du dx . . . . . . V63.0121.006/016, Calculus I (NYU) Section 5.5 Integration by Substitution April 27, 2010 14 / 38
  • 25. A polynomial example Example ∫ 2 Use the substitution u = x + 3 to find (x2 + 3)3 4x dx. . . . . . . V63.0121.006/016, Calculus I (NYU) Section 5.5 Integration by Substitution April 27, 2010 15 / 38
  • 26. A polynomial example Example ∫ 2 Use the substitution u = x + 3 to find (x2 + 3)3 4x dx. Solution If u = x2 + 3, then du = 2x dx, and 4x dx = 2 du. So ∫ ∫ ∫ (x + 3) 4x dx = u 2du = 2 u3 du 2 3 3 1 4 1 2 = u = (x + 3)4 2 2 . . . . . . V63.0121.006/016, Calculus I (NYU) Section 5.5 Integration by Substitution April 27, 2010 15 / 38
  • 27. A polynomial example, by brute force Compare this to multiplying it out: ∫ ∫ ( ) 2 3 (x + 3) 4x dx = x6 + 9x4 + 27x2 + 27 4x dx ∫ ( ) = 4x7 + 36x5 + 108x3 + 108x dx 1 8 = x + 6x6 + 27x4 + 54x2 2 . . . . . . V63.0121.006/016, Calculus I (NYU) Section 5.5 Integration by Substitution April 27, 2010 16 / 38
  • 28. A polynomial example, by brute force Compare this to multiplying it out: ∫ ∫ ( ) 2 3 (x + 3) 4x dx = x6 + 9x4 + 27x2 + 27 4x dx ∫ ( ) = 4x7 + 36x5 + 108x3 + 108x dx 1 8 = x + 6x6 + 27x4 + 54x2 2 Which would you rather do? . . . . . . V63.0121.006/016, Calculus I (NYU) Section 5.5 Integration by Substitution April 27, 2010 16 / 38
  • 29. A polynomial example, by brute force Compare this to multiplying it out: ∫ ∫ ( ) 2 3 (x + 3) 4x dx = x6 + 9x4 + 27x2 + 27 4x dx ∫ ( ) = 4x7 + 36x5 + 108x3 + 108x dx 1 8 = x + 6x6 + 27x4 + 54x2 2 Which would you rather do? It’s a wash for low powers But for higher powers, it’s much easier to do substitution. . . . . . . V63.0121.006/016, Calculus I (NYU) Section 5.5 Integration by Substitution April 27, 2010 16 / 38
  • 30. Compare We have the substitution method, which, when multiplied out, gives ∫ 1 (x2 + 3)3 4x dx = (x2 + 3)4 2 1( 8 ) = x + 12x6 + 54x4 + 108x2 + 81 2 1 81 = x8 + 6x6 + 27x4 + 54x2 + 2 2 and the brute force method ∫ 1 (x2 + 3)3 4x dx = x8 + 6x6 + 27x4 + 54x2 2 Is this a problem? . . . . . . V63.0121.006/016, Calculus I (NYU) Section 5.5 Integration by Substitution April 27, 2010 17 / 38
  • 31. Compare We have the substitution method, which, when multiplied out, gives ∫ 1 (x2 + 3)3 4x dx = (x2 + 3)4 + C 2 1( 8 ) = x + 12x6 + 54x4 + 108x2 + 81 + C 2 1 81 = x8 + 6x6 + 27x4 + 54x2 + +C 2 2 and the brute force method ∫ 1 (x2 + 3)3 4x dx = x8 + 6x6 + 27x4 + 54x2 + C 2 Is this a problem? No, that’s what +C means! . . . . . . V63.0121.006/016, Calculus I (NYU) Section 5.5 Integration by Substitution April 27, 2010 17 / 38
  • 32. A slick example Example ∫ Find tan x dx. . . . . . . V63.0121.006/016, Calculus I (NYU) Section 5.5 Integration by Substitution April 27, 2010 18 / 38
  • 33. A slick example Example ∫ sin x Find tan x dx. (Hint: tan x = ) cos x . . . . . . V63.0121.006/016, Calculus I (NYU) Section 5.5 Integration by Substitution April 27, 2010 18 / 38
  • 34. A slick example Example ∫ sin x Find tan x dx. (Hint: tan x = ) cos x Solution Let u = cos x. Then du = − sin x dx. . . . . . . V63.0121.006/016, Calculus I (NYU) Section 5.5 Integration by Substitution April 27, 2010 18 / 38
  • 35. A slick example Example ∫ sin x Find tan x dx. (Hint: tan x = ) cos x Solution Let u = cos x. Then du = − sin x dx. So ∫ ∫ ∫ sin x 1 tan x dx = dx = − du cos x u . . . . . . V63.0121.006/016, Calculus I (NYU) Section 5.5 Integration by Substitution April 27, 2010 18 / 38
  • 36. A slick example Example ∫ sin x Find tan x dx. (Hint: tan x = ) cos x Solution Let u = cos x. Then du = − sin x dx. So ∫ ∫ ∫ sin x 1 tan x dx = dx = − du cos x u = − ln |u| + C . . . . . . V63.0121.006/016, Calculus I (NYU) Section 5.5 Integration by Substitution April 27, 2010 18 / 38
  • 37. A slick example Example ∫ sin x Find tan x dx. (Hint: tan x = ) cos x Solution Let u = cos x. Then du = − sin x dx. So ∫ ∫ ∫ sin x 1 tan x dx = dx = − du cos x u = − ln |u| + C = − ln | cos x| + C = ln | sec x| + C . . . . . . V63.0121.006/016, Calculus I (NYU) Section 5.5 Integration by Substitution April 27, 2010 18 / 38
  • 38. Can you do it another way? Example ∫ sin x Find tan x dx. (Hint: tan x = ) cos x . . . . . . V63.0121.006/016, Calculus I (NYU) Section 5.5 Integration by Substitution April 27, 2010 19 / 38
  • 39. Can you do it another way? Example ∫ sin x Find tan x dx. (Hint: tan x = ) cos x Solution du Let u = sin x. Then du = cos x dx and so dx = . cos x . . . . . . V63.0121.006/016, Calculus I (NYU) Section 5.5 Integration by Substitution April 27, 2010 19 / 38
  • 40. Can you do it another way? Example ∫ sin x Find tan x dx. (Hint: tan x = ) cos x Solution du Let u = sin x. Then du = cos x dx and so dx = . cos x ∫ ∫ ∫ sin x u du tan x dx = dx = cos x cos x cos x ∫ ∫ ∫ u du u du u du = = = cos2 x 2 1 − sin x 1 − u2 At this point, although it’s possible to proceed, we should probably back up and see if the other way works quicker (it does). . . . . . . V63.0121.006/016, Calculus I (NYU) Section 5.5 Integration by Substitution April 27, 2010 19 / 38
  • 41. For those who really must know all Solution (Continued, with algebra help) ∫ ∫ ∫ ( ) u du 1 1 1 tan x dx = = − du 1 − u2 2 1−u 1+u 1 1 = − ln |1 − u| − ln |1 + u| + C 2 2 1 1 = ln √ + C = ln √ +C (1 − u)(1 + u) 1 − u2 1 = ln + C = ln |sec x| + C |cos x| There are other ways to do it, too. . . . . . . V63.0121.006/016, Calculus I (NYU) Section 5.5 Integration by Substitution April 27, 2010 20 / 38
  • 42. Outline Last Time: The Fundamental Theorem(s) of Calculus Substitution for Indefinite Integrals Theory Examples Substitution for Definite Integrals Theory Examples . . . . . . V63.0121.006/016, Calculus I (NYU) Section 5.5 Integration by Substitution April 27, 2010 21 / 38
  • 43. Substitution for Definite Integrals Theorem (The Substitution Rule for Definite Integrals) If g′ is continuous and f is continuous on the range of u = g(x), then ∫ b ∫ g(b) ′ f(g(x))g (x) dx = f(u) du. a g(a) . . . . . . V63.0121.006/016, Calculus I (NYU) Section 5.5 Integration by Substitution April 27, 2010 22 / 38
  • 44. Substitution for Definite Integrals Theorem (The Substitution Rule for Definite Integrals) If g′ is continuous and f is continuous on the range of u = g(x), then ∫ b ∫ g(b) ′ f(g(x))g (x) dx = f(u) du. a g(a) Why the change in the limits? The integral on the left happens in “x-land” The integral on the right happens in “u-land”, so the limits need to be u-values To get from x to u, apply g . . . . . . V63.0121.006/016, Calculus I (NYU) Section 5.5 Integration by Substitution April 27, 2010 22 / 38
  • 45. Example ∫ π Compute cos2 x sin x dx. 0 . . . . . . V63.0121.006/016, Calculus I (NYU) Section 5.5 Integration by Substitution April 27, 2010 23 / 38
  • 46. Example ∫ π Compute cos2 x sin x dx. 0 Solution (Slow Way) ∫ First compute the indefinite integral cos2 x sin x dx and then evaluate. . . . . . . V63.0121.006/016, Calculus I (NYU) Section 5.5 Integration by Substitution April 27, 2010 23 / 38
  • 47. Example ∫ π Compute cos2 x sin x dx. 0 Solution (Slow Way) ∫ First compute the indefinite integral cos2 x sin x dx and then evaluate. Let u = cos x. Then du = − sin x dx and ∫ ∫ cos x sin x dx = − u2 du 2 = − 1 u3 + C = − 3 cos3 x + C. 3 1 Therefore ∫ π 1 π 1( ) 2 cos x sin x dx = − cos3 x 2 =− (−1)3 − 13 = . 0 3 0 3 3 . . . . . . V63.0121.006/016, Calculus I (NYU) Section 5.5 Integration by Substitution April 27, 2010 23 / 38
  • 48. Definite-ly Quicker Solution (Fast Way) Do both the substitution and the evaluation at the same time. . . . . . . V63.0121.006/016, Calculus I (NYU) Section 5.5 Integration by Substitution April 27, 2010 24 / 38
  • 49. Definite-ly Quicker Solution (Fast Way) Do both the substitution and the evaluation at the same time. Let u = cos x. Then du = − sin x dx, u(0) = 1 and u(π) = −1. . . . . . . V63.0121.006/016, Calculus I (NYU) Section 5.5 Integration by Substitution April 27, 2010 24 / 38
  • 50. Definite-ly Quicker Solution (Fast Way) Do both the substitution and the evaluation at the same time. Let u = cos x. Then du = − sin x dx, u(0) = 1 and u(π) = −1. So ∫ π ∫ −1 ∫ 1 2 cos x sin x dx = −u du = 2 u2 du 0 1 −1 1 3 1 1( ) 2 = u = 1 − (−1) = 3 −1 3 3 . . . . . . V63.0121.006/016, Calculus I (NYU) Section 5.5 Integration by Substitution April 27, 2010 24 / 38
  • 51. Definite-ly Quicker Solution (Fast Way) Do both the substitution and the evaluation at the same time. Let u = cos x. Then du = − sin x dx, u(0) = 1 and u(π) = −1. So ∫ π ∫ −1 ∫ 1 2 cos x sin x dx = −u du = 2 u2 du 0 1 −1 1 3 1 1( ) 2 = u = 1 − (−1) = 3 −1 3 3 The advantage to the “fast way” is that you completely transform the integral into something simpler and don’t have to go back to the original variable (x). But the slow way is just as reliable. . . . . . . V63.0121.006/016, Calculus I (NYU) Section 5.5 Integration by Substitution April 27, 2010 24 / 38
  • 52. An exponential example Example ∫ ln √8 √ 2x Find √ e e2x + 1 dx ln 3 . . . . . . V63.0121.006/016, Calculus I (NYU) Section 5.5 Integration by Substitution April 27, 2010 25 / 38
  • 53. An exponential example Example ∫ ln √8 √ 2x Find √ e e2x + 1 dx ln 3 Solution Let u = e2x , so du = 2e2x dx. We have ∫ √ ∫ ln 8 √ 1 8√ 2x √ e e2x + 1 dx = u + 1 du ln 3 2 3 . . . . . . V63.0121.006/016, Calculus I (NYU) Section 5.5 Integration by Substitution April 27, 2010 25 / 38
  • 54. About those limits Since √ √ 2 e2(ln 3) = eln 3 = eln 3 = 3 we have √ ∫ ln 8 √ ∫ 8√ 2x 1 √ e e2x + 1 dx = u + 1 du ln 3 2 3 . . . . . . V63.0121.006/016, Calculus I (NYU) Section 5.5 Integration by Substitution April 27, 2010 26 / 38
  • 55. An exponential example Example ∫ ln √8 √ 2x Find √ e e2x + 1 dx ln 3 Solution Let u = e2x , so du = 2e2x dx. We have ∫ √ ∫ ln 8 √ 1 8√ 2x √ e e2x + 1 dx = u + 1 du ln 3 2 3 Now let y = u + 1, dy = du. So ∫ 8√ ∫ 9√ ∫ 9 1 1 1 u + 1 du = y dy = y1/2 dy 2 3 2 4 2 4 9 1 2 1 19 = · y3/2 = (27 − 8) = 2 3 4 3 . 3 . . . . . V63.0121.006/016, Calculus I (NYU) Section 5.5 Integration by Substitution April 27, 2010 27 / 38
  • 56. About those fractional powers We have 93/2 = (91/2 )3 = 33 = 27 43/2 = (41/2 )3 = 23 = 8 so ∫ 9 9 1 1 2 3/2 1 19 y1/2 dy = · y = (27 − 8) = 2 4 2 3 4 3 3 . . . . . . V63.0121.006/016, Calculus I (NYU) Section 5.5 Integration by Substitution April 27, 2010 28 / 38
  • 57. An exponential example Example ∫ ln √8 √ 2x Find √ e e2x + 1 dx ln 3 Solution Let u = e2x , so du = 2e2x dx. We have ∫ √ ∫ ln 8 √ 1 8√ 2x √ e e2x + 1 dx = u + 1 du ln 3 2 3 Now let y = u + 1, dy = du. So ∫ 8√ ∫ 9√ ∫ 9 1 1 1 u + 1 du = y dy = y1/2 dy 2 3 2 4 2 4 9 1 2 1 19 = · y3/2 = (27 − 8) = 2 3 4 3 . 3 . . . . . V63.0121.006/016, Calculus I (NYU) Section 5.5 Integration by Substitution April 27, 2010 29 / 38
  • 58. Another way to skin that cat Example ∫ ln √8 √ Find √ e2x e2x + 1 dx ln 3 Solution Let u = e2x + 1, . . . . . . V63.0121.006/016, Calculus I (NYU) Section 5.5 Integration by Substitution April 27, 2010 30 / 38
  • 59. Another way to skin that cat Example ∫ ln √8 √ Find √ e2x e2x + 1 dx ln 3 Solution Let u = e2x + 1,so that du = 2e2x dx. . . . . . . V63.0121.006/016, Calculus I (NYU) Section 5.5 Integration by Substitution April 27, 2010 30 / 38
  • 60. Another way to skin that cat Example ∫ ln √8 √ Find √ e2x e2x + 1 dx ln 3 Solution Let u = e2x + 1,so that du = 2e2x dx. Then ∫ √ ∫ ln 8 √ 1 9√ 2x √ e e2x + 1 dx = u du ln 3 2 4 . . . . . . V63.0121.006/016, Calculus I (NYU) Section 5.5 Integration by Substitution April 27, 2010 30 / 38
  • 61. Another way to skin that cat Example ∫ ln √8 √ Find √ e2x e2x + 1 dx ln 3 Solution Let u = e2x + 1,so that du = 2e2x dx. Then ∫ √ ∫ ln 8 √ 1 9√ 2x √ e e2x + 1 dx = u du ln 3 2 4 9 1 3/2 = u 3 4 . . . . . . V63.0121.006/016, Calculus I (NYU) Section 5.5 Integration by Substitution April 27, 2010 30 / 38
  • 62. Another way to skin that cat Example ∫ ln √8 √ Find √ e2x e2x + 1 dx ln 3 Solution Let u = e2x + 1,so that du = 2e2x dx. Then ∫ √ ∫ ln 8 √ 1 9√ 2x √ e e2x + 1 dx = u du ln 3 2 4 1 3/2 9 = u 3 4 1 19 = (27 − 8) = 3 3 . . . . . . V63.0121.006/016, Calculus I (NYU) Section 5.5 Integration by Substitution April 27, 2010 30 / 38
  • 63. A third skinned cat Example ∫ ln √8 √ 2x Find √ e e2x + 1 dx ln 3 Solution √ Let u = e2x + 1, so that u2 = e2x + 1 . . . . . . V63.0121.006/016, Calculus I (NYU) Section 5.5 Integration by Substitution April 27, 2010 31 / 38
  • 64. A third skinned cat Example ∫ ln √8 √ 2x Find √ e e2x + 1 dx ln 3 Solution √ Let u = e2x + 1, so that u2 = e2x + 1 =⇒ 2u du = 2e2x dx . . . . . . V63.0121.006/016, Calculus I (NYU) Section 5.5 Integration by Substitution April 27, 2010 31 / 38
  • 65. A third skinned cat Example ∫ ln √8 √ 2x Find √ e e2x + 1 dx ln 3 Solution √ Let u = e2x + 1, so that u2 = e2x + 1 =⇒ 2u du = 2e2x dx Thus √ ∫ ln 8 ∫ 3 3 1 19 √ = u · u du = u3 = ln 3 2 3 2 3 . . . . . . V63.0121.006/016, Calculus I (NYU) Section 5.5 Integration by Substitution April 27, 2010 31 / 38
  • 66. A Trigonometric Example Example Find ∫ ( ) ( ) 3π/2 θ5 2 θ cot sec dθ. π 6 6 . . . . . . V63.0121.006/016, Calculus I (NYU) Section 5.5 Integration by Substitution April 27, 2010 32 / 38
  • 67. A Trigonometric Example Example Find ∫ ( ) ( ) 3π/2 θ5 2 θ cot sec dθ. π 6 6 Before we dive in, think about: What “easy” substitutions might help? Which of the trig functions suggests a substitution? . . . . . . V63.0121.006/016, Calculus I (NYU) Section 5.5 Integration by Substitution April 27, 2010 32 / 38
  • 68. Solution θ 1 Let φ = . Then dφ = dθ. 6 6 ∫ 3π/2 ( ) ( ) ∫ π/4 5 θ 2 θ cot sec dθ = 6 cot5 φ sec2 φ dφ π 6 6 π/6 ∫ π/4 sec2 φ dφ =6 π/6 tan5 φ . . . . . . V63.0121.006/016, Calculus I (NYU) Section 5.5 Integration by Substitution April 27, 2010 33 / 38
  • 69. Solution θ 1 Let φ = . Then dφ = dθ. 6 6 ∫ 3π/2 ( ) ( ) ∫ π/4 5 θ 2 θ cot sec dθ = 6 cot5 φ sec2 φ dφ π 6 6 π/6 ∫ π/4 sec2 φ dφ =6 π/6 tan5 φ Now let u = tan φ. So du = sec2 φ dφ, and ∫ π/4 ∫ 1 sec2 φ dφ −5 6 =6 √ u du π/6 tan5 φ 1/ 3 ( ) 1 −4 1 3 =6 − u √ = [9 − 1] = 12. 4 1/ 3 2 . . . . . . V63.0121.006/016, Calculus I (NYU) Section 5.5 Integration by Substitution April 27, 2010 33 / 38
  • 70. The limits explained √ π sin π/4 2/2 tan = =√ =1 4 cos π/4 2/2 π sin π/6 1/2 1 tan = =√ =√ 6 cos π/6 3/2 3 . . . . . . V63.0121.006/016, Calculus I (NYU) Section 5.5 Integration by Substitution April 27, 2010 34 / 38
  • 71. The limits explained √ π sin π/4 2/2 tan = =√ =1 4 cos π/4 2/2 π sin π/6 1/2 1 tan = =√ =√ 6 cos π/6 3/2 3 ( ) √ 1 −4 1 3 [ −4 ]1 3 [ −4 ]1/ 3 6 − u √ = −u √ = u 4 1/ 3 2 1/ 3 2 1 3 [ ] = (3−1/2 )−4 − (1−1/2 )−4 2 3 3 = [32 − 12 ] = (9 − 1) = 12 2 2 . . . . . . V63.0121.006/016, Calculus I (NYU) Section 5.5 Integration by Substitution April 27, 2010 34 / 38
  • 72. Graphs ∫ 3π/2 ( ) ( ) ∫ π/4 θ 2 θ . cot5 sec dθ . 6 cot5 φ sec2 φ dφ π 6 6 π/6 y . y . . . . . θ . . . φ 3π . π ππ . . . 2 64 The areas of these two regions are the same. . . . . . . V63.0121.006/016, Calculus I (NYU) Section 5.5 Integration by Substitution April 27, 2010 35 / 38
  • 73. Graphs ∫ ∫ π/4 1 5 2 −5 . 6 cot φ sec φ dφ . √ 6u du π/6 1/ 3 y . y . . . . . φ . .. u ππ 1 .1 . . .√ 64 3 The areas of these two regions are the same. . . . . . . V63.0121.006/016, Calculus I (NYU) Section 5.5 Integration by Substitution April 27, 2010 36 / 38
  • 74. Final Thoughts Antidifferentiation is a “nonlinear” problem that needs practice, intuition, and perserverance Worksheet in recitation (also to be posted) The whole antidifferentiation story is in Chapter 6 . . . . . . V63.0121.006/016, Calculus I (NYU) Section 5.5 Integration by Substitution April 27, 2010 37 / 38
  • 75. Summary If F is an antiderivative for f, then: ∫ f(g(x))g′ (x) dx = F(g(x)) ∫ b ∫ g(b) ′ f(g(x))g (x) dx = f(u) du = F(g(b)) − F(g(a)) a g(a) . . . . . . V63.0121.006/016, Calculus I (NYU) Section 5.5 Integration by Substitution April 27, 2010 38 / 38